15
$\begingroup$

Let ${\cal F}=\left\{ f:\left[0,1\right]\to\mathbb{R} : \left|f\left(x\right)-f\left(y\right)\right|\le\left|x-y\right|\mbox{ and }{\displaystyle \int_{0}^{1}f\left(x\right)dx=1}\right\}.$ Show that ${\cal F}$ is a compact subset of $C\left(\left[0,1\right]\right)$.

When I am trying to show a set is compact, I usually resort to the every open cover has a finite subcover definition. But if this case, we are dealing with functions. So I am having a difficulty "visualizing" what's going on. Any help or solutions would be appreciated.

Edit: I should mention that we are working with respect to the sup norm.

  • 1
    Perhaps it's not particularly usual for this question, but since you mentioned that you don't know how to visualize $C[0,1]$ with sup-norm, you might have a look [here](http://math.stackexchange.com/questions/148349/open-and-closed-balls-in-ca-b).2012-06-03

1 Answers 1

14

According to Arzelà–Ascoli theorem you only have to show that $\mathcal F$ is

  • equicontinuous, i.e. $(\forall x\in[0,1])(\forall \varepsilon>0)(\exists \delta>0)(\forall f\in\mathcal F) (\forall y) |y-x|<\delta \Rightarrow |f(y)-f(x)|<\varepsilon$;
  • pointwise bounded;
  • closed in $C[0,1]$.

Both equicontinuity and pointwise boundedness follow from the Lipschitz condition $|f(x)-f(y)|\le |x-y|$.

To show pointwise boundedness you can notice that $|f(x)-f(0)|\le |x|=x$, which means $f(0)-x \le f(x) \le f(0)+x.$ If you apply integral $\int_0^1$ to the left inequality, you get $f(0)\le\int_0^1 (x+f(x))\,\mathrm{d}x=\frac32$. Now the right inequality implies $f(x)\le \frac52$ for each $x$. (Thanks to Nate Eldredge, who pointed in his comment, that this was missing in my original answer.)

To show that it is closed in sup-norm, you only have to show that if $f_n$ converges to $f$ uniformly and $f_n\in\mathcal F$, then the limit is in $\mathcal F$. We know that integral behaves well w.r.t. uniform convergence, see this questions. Proof of the fact that the condition $(\forall x,y\in [0,1])|f(x)-f(y)|\le |x-y|$ is preserved by uniform convergence is more-or-less standard. (In fact, in this part we only use pointwise convergence.)

  • 5
    @Galois: It would be a worthwhile and probably enlightening exercise *not* to use the Arzelà-Ascoli theorem and do the proof of (sequential) compactness of $\mathcal{F}$ by hand. If you're getting stuck, look at the proof of the Arzelà-Ascoli theorem and adapt it to the present situation.2012-06-03